4510: Concept Synthesis, FINAL Exam new content only - Cholecystitis, Pancreatitis, Liver Issues, Organ Transplant, Parkinson's, Chronic Neuro Problems (MS, MG)

¡Supera tus tareas y exámenes ahora con Quizwiz!

A client presents with an exacerbation of multiple sclerosis. Which assessment finding is expected? 1. Double vision and nystagmus 2. Change in mental status 3. Tremors upon resting 4. Flaccid paralysis

1. Tremors upon resting are not characteristic of MS; MS is characterized by ***intentional tremors. Spastic paralysis, not flaccid paralysis, is characteristic of MS

What is the mechanism of action for levodopa administration for a client with Parkinson disease? 1. Levodopa promotes the production of acetylcholine 2. Levodopa blocks the effects of acetylcholine 3. Levodopa restores the dopamine levels in the brain 4. Levodopa increases the production of dopamine

3. Levodopa restores the dopamine levels in the brain

While providing mouth care to a patient with late-stage cirrhosis, you note a pungent, sweet, musty smell to the breath. This is known as: A. Metallic Hepatico B. Fetor Hepaticus C. Hepaticoacidosis D. Asterixis

B

A client is hospitalized with severe dyspnea due to ascites. A change in which of the following most likely caused the ascites? A. Decreased interstitial osmotic pressure B. Increased pressure in the portal vein C. Decreased secretion of bile salts D. Increaesed production of serum albumin

B The enlarged cirrhotic liver impinges on the portal system, causing increased hydrostatic pressure from increased pressure in the portal vein, resulting in ascites. Bile salts are not responsible for fluid shifts; increased serum bile results from biliary obstruction, not increased secretion of bile. Interstitial osmotic pressure is unchanged; decreased intravascular osmotic pressure accounts for fluid movement into interstitial spaces. The liver's production of serum albumin is decreased with cirrhosis of the liver.

A patient with late-stage cirrhosis develops portal hypertension. Which of the following options below are complications that can develop from this condition? Select all that apply: A. Increase albumin levels B. Ascites C. Splenomegaly D. Fluid volume deficient E. Esophageal varices

B, C, and E. Portal Hypertension is where the portal vein becomes narrow due to scar tissue in the liver, which is restricting the flow of blood to the liver. Therefore, pressure becomes increased in the portal vein and affects the organs connected via the vein to the liver. The patient may experience

You're performing a head-to-toe assessment on a patient with multiple sclerosis. When you ask the patient to move the head and neck downward the patient reports an "electric shock" sensation that travels down the body. You would report your finding to the doctor that the patient is experiencing: A. Romberg's Sign B. Lhermitte's Sign C. Uhthoff's Sign D. Homan's Sign

B. This finding is known as Lhermitte's Sign.

A patient with Parkinson's Disease has slow movements that affects their swallowing, facial expressions, and ability to coordinate movements. As the nurse you will document the patient has: A. Akinesia B. "Freeze up" tremors C. Bradykinesia D. Pill-rolling

C

Which of the following is NOT a role of the liver? A. Removing hormones from the body B. Producing bile C. Absorbing water D. Producing albumin

C. the intestine is responsible for absorbing water

Which of the following is NOT a common source of transmission for Hepatitis A? Select all that apply: A. Water B. Food C. Semen D. Blood

The answers are C and D. The most common source for transmission of Hepatitis A is water and food.

TRUE or FALSE: A patient with Hepatitis A is contagious about 2 weeks before signs and symptoms appear and 1-3 weeks after the symptoms appear.

True

When caring for a patient with a biliary obstruction, the nurse will anticipate administering which vitamin supplements? (Select all that apply.) Vitamin A Vitamin B Vitamin D Vitamin E Vitamin K

D, A, K, E Biliary obstruction prevents bile from entering the small intestine and thus prevents the absorption of fat-soluble vitamins. Vitamins A, D, E, and K are all fat-soluble and thus would need to be supplemented in a patient with biliary obstruction.

The condition of a patient who has cirrhosis of the liver has deteriorated. Which diagnostic study would best help determine if the patient has developed liver cancer? 1. MRI scanning 2. Serum α-fetoprotein level 3. Ventilation/perfusion scan 4. Abdominal girth measurement

1. Hepatic ultrasonography, CT scan, and MRI scanning are used to screen for and diagnose liver cancer. Serum α-fetoprotein level may be elevated with liver cancer or other liver problems. Ventilation/perfusion scans are used to diagnose pulmonary emboli. Abdominal girth measurement would not differentiate between cirrhosis and liver cancer.

Which statement regarding monoamine oxidase inhibitors is correct? 1. MAO inhibitors are often prescribed to clients with Parkinson disease to reduce tremors 2. Selegiline is a non-selective MAO inhibitor of MAO-B (selective) 3. MAO inhibitors enhance the breakdown of dopamine (No, increase) 4. MAO inhibitors can cause hypotension in some clients

1. MAO inhibitors are often prescribed to clients with Parkinson disease to reduce tremors -Selegiline (L-deprenyl) is a selective, irreversible inhibitor of monoamine oxidase B (MAO-B) at the conventional dose (10 mg/day oral) that is used in the treatment of Parkinson's disease. Must be used in conjunction with Sinemet to reduce tremors -nonspecific MAOIs include Phenoxypropazine and trancylcypromine - MAOIs increase function of the serotonin (5-HT), NE and dopamine systems. -MAOIs cause HTN crisis if there is high tyramine in the system.

Which test is used to diagnose myasthenia gravis? 1. Magnetic resonance imaging (MRI) 2. Edrophonium chloride (Tensilon) test 3. There is no test, the diagnosis is made based on clinical presentation 4. Pyridostigmine test

2 Edrophonium temporarily blocks the breakdown of acetylcholine, thus increasing acetylcholine level in the blood, and relieves weakness in MG. Because of its short duration of action, edrophonium is the drug of choice for diagnosing myasthenia gravis. It's also used to differentiate myasthenia gravis from cholinergic toxicity. Pyridostigmine serves primarily as an adjunct in treating severe anticholinergic toxicity; it's also an antiglaucoma agent and a miotic. Tensilon test: · Injection of IV edrophonium chloride shows rapid muscle strength improvement in pt with myasthenia gravis. Positive test · Can also diagnoses a cholinergic crisis, there will be a worsening of s&s in pt with anticholinergic crisis; administer ATROPINE as edrophonium antidote.

What is the mechanism of action of the medication pyridostigmine for the treatment of myasthenia gravis? 1. Pyridostigmine directly removes acetylcholine receptor (AChR) antibodies, preventing the descruction of acetylcholine (ACh) receptors 2. Pyridostigmine inhibits the acetylcholinesterase enzyme and keeps it from breaking down acetylcholine (ACh), making ACh more available for the remaining receptors 3. Pyridostigmine inhibits the immune system, preventing the destruction of acetylcholine (ACh) receptors. 4. Pyridostigmine has an uncertain mechanism of action, but effects can be seen in less than a week.

2. Peripheral acetylcholinesterase inhibitor, which enhances transmission at the neuromuscular junction. Inhibiting acetylcholinesterase prolongs the action of ACh and improves impulse transmission at the neuromuscular junction. Pyridostigmine (Mestinon) is the most successful drug of this group in the long-term treatment of MG. (Lewis 1377)

Which cytokine is used in the treatment of multiple sclerosis? 1. Erythropoietin 2. Interferon beta 3. Interleukin-2 4. Colony-stimulating factor

2. Treatment of MS begins with use of immunomodulator drugs to modify disease progression and prevent relapses. These drugs include (1) interferon β-1a (Rebif, Plegridy, Avonex), (2) interferon β-1b (Betaseron, Extavia), and (3) glatiramer acetate (Copaxone, Glatopa).

What education would be included for a client regarding prevention of contracting Hepatitis B? 1. Prevention of constipation 2. Practicing safe sex 3. Avoiding raw shellfish 4. Limit hepatotoxic medications

2. Contracting hepatitis B through blood transfusion can be prevented by screening donors and testing the blood. Hepatitis B can be transmitted via contaminated body fluids such as semen, saliva, and urine. Having multiple sexual partners increases the risk. A monogamous sexual relationship with an infection-free individual eliminates the risk. Preventing constipation is not related to limiting the risk for contracting hepatitis Avoiding shellfish in the diet limits the risk for contracting hepatitis Limiting hepatotoxic drug therapy does not prevent transmission of hepatitis B.

Which assessment finding is common for both myasthenic and cholinergic crises in a client with a history of myasthenia gravis? 1. Excessive salivation 2. Difficulty breathing 3. Excessive diarrhea 4. Moderate abdominal cramping

2. Difficulty breathing (respiratory failure) Both myasthenic crisis and cholinergic crisis may cause bronchospasm with wheezing, bronchorrhea, respiratory failure, diaphoresis, and cyanosis Cholinergic crisis: excessive stimulation at NMS by ACh, leads to overdrive in cholinergic response; pupil constriction, miosis, increased salivation and tears, blurred vision, muscle fasciculation (twitch), diarrhea and abdominal cramping, vomiting, respiratory failure. · Cause: too much anticholinesterase medication leading to overdrive of Ach axn Myasthenic crisis: Low to no stimulation at NMS by Ach; leads to severe muscle weakness, pupil dilation, increase BP/HR, risk for aspiration, bowel and bladder incontinence, negative gag or cough reflex, respiratory failure. · Cause: not enough anticholinesterase medication, acute exacerbation from stress, respiratory infection

Which care measure is a priority for a patient with multiple sclerosis (MS)? 1. Vigilant infection control and adherence to standard precautions 2. Careful monitoring of neurologic assessment and frequent reorientation 3. Maintenance of a calorie count and hourly assessment of intake and output 4. Assessment of blood pressure and monitoring for signs of orthostatic hypotension

2. Infection control is a priority in the care of patients with MS because infection is the most common cause of an exacerbation of the disease. Decreases in cognitive function are less likely, and MS does not typically result in malnutrition, hypotension, or fluid volume excess or deficit.

The nurse observes a 74-yr-old man with Parkinson's disease rocking side to side while sitting in the chair. Which action by the nurse is most appropriate? 1. Provide the patient with diversional activities. 2. Document the activity in the patient's health record. 3. Take the patient's blood pressure sitting and standing. 4. Ask if the patient is feeling either anxious or depressed.

2. Patients with Parkinson's disease are taught to rock from side to side to stimulate balance mechanisms and decrease akinesia.

The patient with right upper quadrant abdominal pain has an abdominal ultrasound that reveals cholelithiasis. What is the nurse's priority? 1. Prevent all oral intake. 2. Control abdominal pain. 3. Provide enteral feedings. 4. Avoid dietary cholesterol.

2. Patients with cholelithiasis can have severe pain, so controlling pain is important until the problem can be treated. NPO status may be needed if the patient will have surgery but will not be used for all patients with cholelithiasis. Patients with pancreatitis may be NPO. Enteral feedings should not be needed, and avoiding dietary cholesterol is not used to treat cholelithiasis.

The nurse is caring for a group of patients on a medical unit. After receiving report, which patient should the nurse see first? 1. A 42-yr-old patient with multiple sclerosis who was admitted with sepsis 2. A 72-yr-old patient with Parkinson's disease who has aspiration pneumonia. 3. A 38-yr-old patient with myasthenia gravis who declined prescribed medications 4. A 45-yr-old patient with amyotrophic lateral sclerosis who refuses enteral feedings.

2. Patients with myasthenia gravis who discontinue pyridostigmine (Mestinon) will develop myasthenic crisis. Myasthenia crisis results in severe muscle weakness and can lead to a respiratory arrest.

A patient admitted with diabetes, malnutrition, osteomyelitis, and chronic alcohol use has a serum amylase level of 480 U/L and a serum lipase level of 610 U/L. Which diagnosis does the nurse expect? 1. Starvation 2. Pancreatitis 3. Systemic sepsis 4. Diabetic ketoacidosis

2. The patient with chronic alcohol use could develop pancreatitis as a complication, which would increase the serum amylase (normal, 30 to 122 U/L) and serum lipase (normal, 31 to 186 U/L) levels as shown.

When establishing a diagnosis of multiple sclerosis (MS), which diagnostic tests should the nurse expect? (Select all that apply.) 1. EEG 2. ECG 3. CT scan 4. Carotid duplex scan 5. Evoked response testing 6. Cerebrospinal fluid analysis

3, 5, 6 No definitive diagnostic test exists for MS. Along with history and physical examination, CT scan, evoked response testing, cerebrospinal fluid analysis, and MRI are used to establish a diagnosis of MS. EEG, ECG, and carotid duplex scan are not used to diagnose MS.

A nurse on the progressive unit has a four client assignment. One client has a diagnosis of Parkinson disease, while another has a diagnosis of myasthenia gravis. Which priority problem should be assessed in both patients? 1. Fatigue 2. Dysarthria 3. Dysphagia 4. Cogwheel gait

3. Difficulty swallowing (dysphagia) is a manifestation of both neurologic disorders. With Parkinson disease there is a progressive loss of spontaneity of movement, including swallowing, related to degeneration of the dopamine-producing neurons in the substantia nigra of the midbrain. With myasthenia gravis there is a decreased number of acetylcholine (Ach) receptor sites at the neuromuscular junction, which interferes with muscle contraction, impairing muscles involved in chewing, swallowing, speaking, and breathing. A cogwheel gait is associated with Parkinson disease, not myasthenia gravis. Impaired cognition is associated with Parkinson disease, not myasthenia gravis. Nonintention tremors are associated with Parkinson disease, not myasthenia gravis. The nonintention tremors associated with Parkinson disease result from the loss of the inhibitory influence of dopamine in the basal ganglia, which interferes with the feedback circuit within the cerebral cortex.

Which of the following characteristics would be present in a client with Parkinson disease? 1. The client limps with ambulation ("short leg gait- fromarthritis or fracture) 2. There is increased hip and knee flexion upon taking a step ("steppage gait"-from peroneal nerve injury and paralyzed dorsiflexor muscles) 3. The trunk and knees flex when the body is rigid ("festinating gait"- from PD) 4. The client is uncoordinated, and makes jerkey, cross-knee movements with ambulation ("spastic gait"- CP and hemiplegia)

3. ("festinating gait"- from PD) 1. ("short leg gait- fromarthritis or fracture) 2.("steppage gait"-from peroneal nerve injury and paralyzed dorsiflexor muscles) 4. ("spastic gait"- CP and hemiplegia)

The nurse teaches a patient about cholestyramine to reduce pruritus caused by gallbladder disease. Which statement indicates understanding of the instructions? 1. "This medication will help me digest fats and fat-soluble vitamins." 2. "I will apply the medicated lotion sparingly to the areas where I itch." 3. "The medication is a powder and needs to be mixed with milk or juice." 4. "I should take this medication on an empty stomach at the same time each day."

3. For treatment of pruritus, cholestyramine may provide relief. This is a resin that binds bile salts in the intestine, increasing their excretion in the feces. Cholestyramine is in powder form and should be mixed with milk or juice before oral administration.

A patient with type 2 diabetes and chronic hepatitis C asks the nurse if it would be acceptable to take silymarin (milk thistle) to help minimize liver damage. The nurse responds based on what knowledge? 1. Milk thistle may affect liver enzymes and thus alter drug metabolism. 2. Milk thistle is generally safe in recommended doses for up to 10 years. 3. There is unclear scientific evidence for the use of milk thistle in treating cirrhosis. 4. Milk thistle may increase serum glucose levels and is thus contraindicated in diabetes.

3. Scientific evidence indicates there is no real benefit from milk thistle to protect liver cells from toxic damage in the treatment of chronic hepatitis C. Milk thistle does affect liver enzymes and thus could alter drug metabolism. Therefore, patients will need to be monitored for drug interactions. It is generally well tolerated. It may lower, not elevate, blood glucose levels.

Which problem is the nursing priority when caring for a patient with myasthenia gravis (MG)? 1. Acute confusion 2. Bowel incontinence 3. Activity intolerance 4. Disturbed sleep pattern

3. The primary feature of MG is fluctuating weakness of skeletal muscle. Bowel incontinence and confusion are unlikely signs of MG. Although sleep disturbance is likely, activity intolerance is of primary concern.

Which is an expected finding in a client who presents with symptoms from myasthenia gravis? 1. The symptoms are worse first thing in the morning 2. There are intermittent periods of hyperactivity 3. There is an improvement in muscle weakness after the client has rested There is a slow, insidious onset of muscle weakness

3. There is an improvement in muscle weakness after the client has rested

Which clinical finding is expected for a client with Parkinson disease? 1. Tonic-clonic seizures 2. Tardive Dyskinesia 3. Muscle flaccidity 4. Flattened affect

4. Muscle rigidity, blank facial expression (masklike), unintentional resting tremors.

When planning care for a patient with cirrhosis, the nurse will give highest priority to which nursing diagnosis? 1. Fluid imbalance 2. Impaired tissue integrity 3. Impaired nutritional status 4. Ineffective breathing pattern

4. Although all these nursing diagnoses are appropriate and important in the care of a patient with cirrhosis, airway and breathing are always the highest priorities.

A patient with a diagnosis of Parkinson's disease (PD) is admitted to a long-term care facility. Which action should the health care team take to promote adequate nutrition for this patient? 1. Administer multivitamins every morning and with each meal. 2. Provide a diet that is low in complex carbohydrates and high in protein. 3. Give the patient with a pureed diet that is high in potassium and low in sodium. 4. Provide small, frequent meals throughout the day that are easy to chew and swallow.

4. Nutritional support is a priority in the care of persons with PD. Patients may benefit from smaller, more frequent meals that are easy to chew and swallow. Multivitamins are not necessary at each meal. Vitamin and protein intake must be monitored to prevent interactions with medications. Introducing a minced or pureed diet is likely premature, and a low carbohydrate diet is not indicated.

A spouse of a husband who has Parkinson's Disease explains to you that her husband experiences episodes while walking where he freezes and can't move. She asks what can be done to help with these types of episodes to prevent injury. Select all the options that are correct: A. Have the husband try to change direction of movement by moving in the opposite direction when the freeze ups occur. B. Use a cane with a laser point while walking. C. Have the husband try to push through the freeze ups. D. Encourage the husband to consciously lift the legs while walking (as with marching).

A, B, D.

Select all the signs and symptoms below that can present in myasthenia gravis: A. Respiratory failure B. Increased salivation C. Diplopia D. Ptosis E. Slurred speech F. Restlessness G. Mask-like appearance of looking sleepy H. Difficulty swallowing

A, C, D, E, F , G, and H. (restlessness is from hypoxia, which is experienced with respiratory failure)

Your patient with cirrhosis has severe splenomegaly. As the nurse you will make it priority to monitor the patient for signs and symptoms of? Select all that apply: A. Thrombocytopenia B. Vision changes C. Increased PT/INR D. Leukopenia

A, C, and D. A patient with an enlarged spleen (splenomegaly) due to cirrhosis can experience thrombocytopenia (low platelet count), increased PT/INR (means it takes the patient a long time to stop bleeding), and leukopenia (low white blood cells). The spleen stores platelets and WBCs. An enlarged spleen can develop due to portal hypertension, which causes the platelets and WBCs to become stuck inside the spleen due to the increased pressure in the hepatic vein (hence lowering the count and the body's access to these important cells for survival).

You're providing an in-service to new nurse graduates about esophageal varices in patients with cirrhosis. You ask the graduates to list activities that should be avoided by a patient with this condition. Which activities listed are correct: Select all that apply A. Excessive coughing B. Sleeping on the back C. Drinking juice D. Alcohol consumption E. Straining during a bowel movement F. Vomiting

A, D, E, and F. Esophageal varices are dilated vessels that are connected from the throat to the stomach. They can become enlarged due to portal hypertension in cirrhosis and can rupture (this is a medical emergency). The patient should avoid activities that could rupture these vessels, such as excessive cough, vomiting, drinking alcohol, and constipation (straining increases thoracic pressure.)

The nurse knows that a client with cirrhosis of the liver will be prescribed neomycin due to which abnormal laboratory test? A. White blood cell count B. Ammonia level C. Culture and sensitivity D. Alanine aminotransferase (ALT) level

A. Ammonia level

You're caring for a patient with Parkinson's Disease that has tremors. Select the option that is INCORRECT about tremors experienced in this disease: A. The tremors are most likely to occur with purposeful movements. B. A common term used to describe the tremors in the hands and fingers is called "pill-rolling". C. Tremors are one of the most common signs and symptoms in Parkinson's Disease. D. Tremors in this disease can occur in the hands, fingers, arms, legs and even the lips and tongue.

A. This option is the only one that is INCORRECT. Tremors in Parkinson's Disease tend to occurs at rest and will actually improve with movement. (MS may be charcterized by tremor with purposeful movement)

Select all the TRUE statements about the pathophysiology of multiple sclerosis: A. "The dendrites on the neuron are overstimulated leading to the destruction of the axon." B. "The myelin sheath is damaged along the axon." C. "This disease affects the insulating structure found on the neuron in the central nervous system." D. "The dopaminergic neurons in the part of the brain called substantia nigra have started to die."

B and C. In multiple sclerosis the myelin sheath is damaged. MS affects the CNS (central nervous system) and when the myelin sheath becomes damaged it leads to a decrease in nerve transmission.

The physician orders Lactulose 30 mL by mouth per day for a patient with cirrhosis. What findings below demonstrates the medication is working effectively? Select all that apply: A. Decrease albumin levels B. Decrease in Fetor Hepaticus C. Patient is stuporous. D. Decreased ammonia blood level E. Presence of asterixis

B and D. A patient with cirrhosis may experience a complication called hepatic encephalopathy. This will cause the patient to become confused (they may enter into a coma), have pungent, musty smelling breath (fetor hepaticus), asterixis (involuntary flapping of the hands) etc. This is due to the buildup of ammonia in the blood, which affects the brain. Lactulose can be prescribed to help decrease the ammonia levels. Therefore, if the medication is working properly to decrease the level of ammonia the patient would have improving mental status (NOT stuporous), decreased ammonia blood level, decreasing or absence of asterixis, and decreased ammonia blood level.

You're providing diet education to a patient with Parkinson's Disease. Which statement below demonstrates the patient understood your teaching? Select all that apply: A. "I will limit foods high in fiber like fruits and vegetables in my diet." B. "I will be sure to drink 2 Liter of fluid per day." C. "It is very common for me to experience diarrhea with this disease." D. "I will avoid taking Carbidopa/Levodopa with a protein rich meal."

B and D. Constipation (not diarrhea) is a common symptom with Parkinson's Disease. Therefore, the patient should be vigilant about preventing constipation by EATING foods high in fiber like fruits/vegetable and drinking 2 L of fluid per day (unless contraindicated). In addition, diet teaching should be included with the medication Carbidopa/Levodopa. The patient should NOT take this medication with a protein rich meal because levodopa competes with protein in the small intestine (hence decreasing it absorption).

You're providing free education to a local community group about the signs and symptoms of Parkinson's Disease. Select all the signs and symptoms a patient could experience with this disease: A. Increased Salivation B. Loss of smell C. Constipation D. Tremors with purposeful movement E. Shuffling of gait F. Freezing of extremities G. Euphoria H. Coordination issues

B, C, E, F, H. These are all signs and symptoms experienced with PD (they vary among patients). There is NOT increased salivation (although drooling occurs...this is due to the decreased ability to swallow). There are tremors at REST (not purposeful movement) along with depression rather the euphoria.

You are receiving shift report on a patient with cirrhosis. The nurse tells you the patient's bilirubin levels are very high. Based on this, what assessment findings may you expect to find during your head-to-toe assessment? Select all that apply: A. Frothy light-colored urine B. Dark brown urine C. Yellowing of the sclera D. Dark brown stool E. Jaundice of the skin F. Bluish mucous membranes

B, C, and E. High bilirubin levels are because the hepatocytes are no longer able to properly conjugate the bilirubin because they are damaged. This causes bilirubin to leak into the blood and urine (rather than entering the bile and being excreted in the stool). Therefore, the bilirubin stays in the blood and will enter the urine. This will cause the patient to experience yellowing of the skin, sclera of the eyes, and mucous membranes ("jaundice") and have dark brown urine. The stools would be CLAY-COLORED not dark brown (remember bilirubin normally gives stool it brown color but it will be absent).

During your discharge teaching to a patient with multiple sclerosis, you educate the patient on how to avoid increasing symptoms and relapses. You tell the patient to avoid: A. Cold temperatures B. Infection C. Overexertion D. Salt F. Stress

B, C, and F. The patient should also avoid extreme heat, which can increase symptoms.

Which medications below can help treat muscle spasms in a patient with multiple sclerosis? Select all that apply: A. Propranolol B. Isoniazid C. Baclofen D. Diazepam E. Modafinil

C and D. These medications treat muscle spasms in patients with MS.

>Which education is necessary regarding the diet of a client with hepatic encephalopathy and ascites? A. High fat B. High sodium C. Moderate protein D. Low carbohydrates

C. https://quizlet.com/410758161/eaq-liver-diseases-flash-cards/ A low sodium intake controls fluid retention and edema and, consequently, ascites. Vitamins help to repair long-standing nutritional deficits associated with cirrhosis of the liver. A moderate-protein diet reduces formation of ammonia, which must be degraded by the liver. High fat intake is avoided because of related cardiovascular risks and the demand for bile that the liver may not be capable of meeting. High, not low, carbohydrate intake is necessary to meet energy requirements for tissue regeneration.

A client with cirrhosis of the liver presents to the emergency department with slurred speech, altered level of consciousness, drowsiness, and a flapping tremor. What changes in diet will be necessary for this client? A. No protein B. Low protein C. Moderate protein D. High protein

C. Control of hepatic encephalopathy also involves treatment of precipitating causes (Table 43.10). This includes lowering dietary protein intake, preventing and controlling GI bleeds, and, in the case of a bleed, removing the blood promptly from the GI tract to decrease the protein accumulation in the gut. Protein restriction may be needed for some patients right after a severe flare of symptoms (i.e., episodic hepatic encephalopathy). However, protein restriction is rarely needed in patients with cirrhosis and persistent hepatic encephalopathy. For many, malnutrition is a more serious clinical problem than hepatic encephalopathy

A 45 year old male has cirrhosis. The patient reports concern about the development of enlarged breast tissue. You explain to the patient that this is happening because? A. The liver cells are removing too much estrogen from the body which causes the testicles to produce excessive amounts of estrogen, and this leads to gynecomastia. B. The liver is producing too much estrogen due to the damage to the liver cells, which causes the level to increase in the body, and this leads to gynecomastia. C. The liver cells are failing to recycle estrogen into testosterone, which leads to gynecomastia. D. The liver cells are failing to remove the hormone estrogen properly from the body, which causes the level to increase in the body, and this leads to gynecomastia.

D

Which assessment finding would be present in a client with a diagnosis of hepatic encephalopathy? A. Melena B. Hyperlipidemia C. Hyperglycemia D. Asterixis and fetor hepaticus

D. Clinical manifestations of Hepatic encephalopathy: o Changes in neurologic and mental responsiveness o Impaired consciousness o Inappropriate behavior o Sleep disturbances, trouble concentrating, coma o *Asterixis o Impairment in writing o *Fetor hepaticus

client presents with a cardiac dysrhythmia and is prescribed lidocaine. The client has cirrhosis of the liver. What alteration to the lidocaine dosage will be necessary? A. A normal dosage can be used because the other organs will compensate for the decreased liver function B. An equal dosage can be given but the administration rate will need to be increased. C. A higher than usual dosage will be needed because the medication will be metabolized at a higher rate. D. A lower than usual dosage will be needed because the medication will be metabolized at a lower rate.

D. https://quizlet.com/534617872/pharm-exam-2-flash-cards/ A client with a history of cirrhosis of the liver develops heart failure. When ventricular bigeminy develops, the provider orders lidocaine. What alterations in lidocaine dosages does the nurse anticipate? Lower because the drug is metabolized at a diminished rate. The client has heart failure, which causes liver congestion, further compromising liver function; therefore, less than the usual adult dose will be prescribed because the liver will not be able to break down lidocaine as effectively as necessary. A dose higher to compensate for the impaired liver function increases the concentration of lidocaine in the blood, leading to toxicity. Lidocaine is metabolized by the liver; other organs cannot assist in the process. This may be life threatening because the client cannot metabolize lidocaine at the required rate, and toxicity may resul

________ reside in the liver and help remove bacteria, debris, and old red blood cells. A. Hepatocytes B. Langerhan cells C. Enterocytes D. Kupffer cells

D. Kupffer cells perform this function and are one of the two types of cells found in the liver lobules (the functional units of the liver). These cells play a role in helping the hepatocytes turn parts of the old red blood cells into bilirubin.

. Which condition is NOT a known cause of cirrhosis? A. Obesity B. Alcohol consumption C. Blockage of the bile duct D. Hepatitis C E. All are known causes of cirrhosis

E.

True or False: Multiple Sclerosis tends to affect men more than women and occurs during the ages of 50-70 years.

False: MS affects WOMEN more than men and shows up during the ages of 20-40 years.

True or False: Patients with multiple sclerosis have different signs and symptoms because this disease can affect various areas of the peripheral nervous system.

False: Yes, patients with MS have different signs and symptoms because lesions can present at different locations in the CENTRAL NERVOUS SYSTEM....hence the brain and spinal cord (not the peripheral nervous system).

A client with fulminant liver failure presents with ecchymosis on the face and bleeding gums. Which vitamin would be included in the client's plan of care? A. Folic acid B. Vitamin C C. Vitamin K D. Vitamin A

K

Cancer can be a cause of what type of pancreatitis? _____________

Obstructive chronic pancreatitis

What is a postoperative complication that is specific to a client with acute pancreatitis that underwent a subtotal pancreatectomy due to an obstructed pancreatic duct? a. Respiratory distress b. Cholecystitis c. Constipation d. Paralytic ileus

Paralytic ileus Any time a patient undergoes surgery that manipulates the bowels, we expect delayed gastric emptying/constipation. We suspect a paralytic ileus if the patient does not have a bowel movement after about two days, and this needs to be addressed. Unless the patient has other comorbidities or complications during surgery, respiratory distress isn't usually a problem.

How is Hepatitis E transmitted? A. Fecal-oral B. Percutaneous C. Mucosal D. Body fluids

The answer is A.

You're preparing to help the neurologist with conducting a Tensilon test. Which antidote will you have on hand in case of an emergency? A. Atropine B. Protamine sulfate C. Narcan D. Leucovorin

The answer is A. Atropine will help reverse the effects of the drug given during a Tensilon test, which is Edrophonium, in case an emergency arises. Edrophonium is a short-acting cholinergic drug, while atropine is an anticholinergic.

A patient is admitted with hepatic encephalopathy secondary to cirrhosis. Which meal option selection below should be avoided with this patient? A. Beef tips and broccoli rabe B. Pasta noodles and bread C. Cucumber sandwich with a side of grapes D. Fresh salad with chopped water chestnuts

The answer is A. Patients who are experiencing hepatic encephalopathy are having issues with toxin build up in the body, specifically ammonia. Remember that ammonia is the byproduct of protein breakdown, and normally the liver can take the ammonia from the protein breakdown and turn it into urea (but if the cirrhosis is severe enough this can't happen). Therefore, the patient should consume foods LOW in protein until the encephalopathy subsides. Option A is very high in protein while the others are low in protein. Remember meats, legumes, eggs, broccoli rabe, certain grains etc. are high in protein.

A patient with viral Hepatitis states their flu-like symptoms have subsided. However, they now have yellowing of the skin and sclera along with dark urine. Based on this finding, this is what phase of Hepatitis? A. Icteric B. Posticteric C. Preicteric D. Convalescent

The answer is A. The Preicteric (prodromal) Phase: flulike symptoms...joint pain, fatigue, nausea vomiting, abdominal pain change in taste, liver enzymes and bilirubin increasing....Icteric Phase: decrease in the flu-like symptoms but will have jaundice and dark urine (buildup of bilirubin) yellowing of skin and white part of the eyeball, clay-colored stool (bilirubin not going to stool to give it's normal brown color) enlarged liver and pain in this area....Posticteric (convalescent) Phase: jaundice and dark urine start to subside and stool returns to normal brown color, liver enzymes and bilirubin decrease to normal

A patient with Parkinson Disease is experiencing weight loss due to difficulty chewing and swallowing. Which meal option below is the best for this patient? A. Scrambled eggs with a side of cottage cheese B. Grilled cheese with apple slices C. Baked chicken with bacon slices D. Tacos with refried beans

The answer is A. The patient should have a diet of soft foods that are easy to swallow and chew. Option A is the only option that meets that specification.

A patient with multiple sclerosis has issues with completely emptying the bladder. The physician orders the patient to take ___________, which will help with bladder emptying. A. Bethanechol B. Oxybutynin C. Avonex D. Amantadine

The answer is A. This medication is a cholinergic medication that will help with bladder emptying.

A physician orders a patient to take Benztropine (Cogentin). The patient has never taken this medication before and is due to take the first dose at 1000. What statement by the patient requires you to hold the dose and notify the physician? A. "I forgot to tell the doctor I take eye drops for my glaucoma." B. "I had a PET scan last week." C. "I take aspirin once day." D. "My hands are experiencing tremors at rest."

The answer is A. This medication is contraindicated for patients with glaucoma.

A patient with Hepatitis is extremely confused. The patient is diagnosed with Hepatic Encephalopathy. What lab result would correlate with this mental status change? A. Ammonia 100 mcg/dL B. Bilirubin 7 mg/dL C. ALT 56 U/L D. AST 10 U/L

The answer is A. When ammonia levels become high (normal 15-45 mcg/dL) it affects brain function. Therefore, the nurse would see mental status changes in a patient with this ammonia level.

Myasthenia gravis occurs when antibodies attack the __________ receptors at the neuromuscular junction leading to ____________. A. metabotropic; muscle weakness B. nicotinic acetylcholine; muscle weakness C. dopaminergic adrenergic; muscle contraction D. nicotinic adrenergic; muscle contraction

The answer is B. In myasthenia gravis, either the nicotinic acetylcholine receptors are attacked by antibodies created by the immune system (hence why this disease is considered autoimmune) or antibodies are inhibiting the function of muscle-specific kinase (which is a receptor tyrosine kinase that helps with maintaining and building the neuromuscular junction). Either way this leads to the neurotransmitter acetylcholine from being able to communicate with the muscle fiber to make it contract.

Which patient below is at MOST risk for developing a complication related to a Hepatitis E infection? A. A 45-year-old male with diabetes. B. A 26-year-old female in the 3rd trimester of pregnancy. C. A 12-year-old female with a ventricle septal defect. D. A 63-year-old male with cardiovascular disease.

The answer is B. Patients who are in the 3rd trimester of pregnancy are at a HIGH risk of developing a complication related to a Hepatitis E infection.

You're a home health nurse providing care to a patient with myasthenia gravis. Today you plan on helping the patient with bathing and exercising. When would be the best time to visit the patient to help these tasks? A. Mid-afternoon B. Morning C. Evening D. Before bedtime

The answer is B. Patients with MG tend to have the best muscle strength in the morning after sleeping or resting rather than at the end of the day....the muscles are tired from being used and the muscle become weaker as the day progresses etc. Therefore any rigorous activities are best performed in the morning or after the patient has rested.

You're providing teaching to a group of patients with myasthenia gravis. Which of the following is not a treatment option for this condition? A. Plasmapheresis B. Cholinesterase medications C. Thymectomy D. Corticosteroids

The answer is B. These medications are not used to treat MG, but ANTIcholinesterase medications (like Pyridostigmine) are used to treat this condition.

Your patient is scheduled for a lumbar puncture to help diagnose multiple sclerosis. The patient wants clarification about what will be found in the cerebrospinal fluid during the lumbar puncture to confirm the diagnosis of MS. You explain that ____________ will be present in the fluid if MS is present. A. high amounts of IgM B. oligoclonal bands C. low amounts of WBC D. oblong red blood cells and glucose

The answer is B. These specific proteins, oligoclonal bands, which are immunoglobulins will be found in the CSF. This demonstrates there is inflammation in the CNS and is a common finding in multiple sclerosis.

While assessing a patient with Parkinson's Disease, you note the patient's arms slightly jerk as you passively move them toward the patient's body. This is known as: A. Lead Pipe Rigidity B. Cogwheel Rigidity C. Pronate Rigidity D. Flexor Rigidity

The answer is B. This is known as cogwheel rigidity, and occurs when the arms are passively moved, which will cause them to jerk slightly.

A patient is prescribed to take Carbidopa/Levodopa (Sinemet). As the nurse you know that which statement is incorrect about this medication: A. It can take up to 3 weeks for the patient to notice a decrease in signs and symptoms when beginning treatment with this medication. B. Body fluids can turn a dark color and stain clothes. C. This medication is most commonly prescribed with a vitamin B6 supplement. D. Carbidopa helps to prevent Levodopa from being broken down in the blood before it enters the brain. Hence, levodopa is able to enter the brain.

The answer is C. All the other options are CORRECT about this medication. However, the patient should avoid foods and supplements high in vitamin B 6 because it decreases the effectiveness of this medication.

During your morning assessment of a patient with cirrhosis, you note the patient is disoriented to person and place. In addition while assessing the upper extremities, the patient's hands demonstrate a flapping motion. What lab result would explain these abnormal assessment findings? A. Decreased magnesium level B. Increased calcium level C. Increased ammonia level D. Increased creatinine level

The answer is C. Based on the assessment findings and the fact the patient has cirrhosis, the patient is experiencing hepatic encephalopathy. This is due to the buildup of toxins in the blood, specifically ammonia. The flapping motion of the hands is called "asterixis". Therefore, an increased ammonia level would confirm these abnormal assessment findings.

You're patient with Parkinson's Disease has been taking Carbidopa/Levodopa for several years. The patient reports that his signs and symptoms actually become worse before the next dose of medication is due. As the nurse, you know what medication can be prescribed with this medication to help decrease this for happening? A. Anticholinergic (Benztropine) B. Dopamine agonists (Ropinirole) C. COMT Inhibitor (Entacapone) D: Beta blockers (Metoprolol)

The answer is C. Entacapone "Comtan" (is a catechol-O-methyltransferase inhibitors) and is used with levodopa/carbidopa to prevent the "wearing off" of the drug before the next dose is due. It blocks the COMT enzyme that will break down the levodopa in the blood to allow it to last longer.

A patient is receiving Interferon Beta for treatment of multiple sclerosis. As the nurse you will stress the importance of? A. Physical exercise to improve fatigue B. Low fat diet C. Hand hygiene and avoiding infection D. Reporting ideation of suicide

The answer is C. Interferon Beta decreases the number of relapses of symptoms in MS patients by decreasing the immune system response, but it lowers the white blood cells count. Hence, there is a risk of infection. It is very important the nurse stresses the importance of hand hygiene and avoiding infection.

Which meal option would be the most appropriate for a patient with myasthenia gravis? A. Roasted potatoes and cubed steak B. Hamburger with baked fries C. Clam chowder with mashed potatoes D. Fresh veggie tray with sliced cheese cubes

The answer is C. Patients with MG have weak muscles and this can include the muscles that are used for chewing and swallowing. The patient should choose meal options that require the least amount of chewing and that are easy to swallow. Option C is a thick type of soup and the mashed potatoes are soft....both are very easy to eat and swallow compared to the other options.

A patient with myasthenia gravis will be eating lunch at 1200. It is now 1000 and the patient is scheduled to take Pyridostigmine. At what time should you administer this medication so the patient will have the maximum benefit of this medication? A. As soon as possible B. 1 hour after the patient has eaten (at 1300) C. 1 hour before the patient eats (at 1100) D. at 1200 right before the patient eats

The answer is C. Pyridostigmine is an anticholinesterase medication that will help improve muscle strength. It is important the patient has maximum muscle strength while eating for the chewing and swallowing process. Therefore, the medication should be given 1 hour before the patient eats because this medication peaks (has the maximum effect) at approximately 1 hour after administration. How does the medication improve muscle strength? It does this by preventing the breakdown of acetylcholine. Remember the nicotinic acetylcholine receptors are damaged and the patient needs as much acetylcholine as possible to prevent muscle weakness. Therefore, this medication will allow more acetylcholine to be used...hence improving muscle strength.

You're educating a patient about the pathophysiology of myasthenia gravis. While explaining the involvement of the thymus gland, the patient asks you where the thymus gland is located. You state it is located? A. behind the thyroid gland B. within the adrenal glands C. behind the sternum in between the lungs D. anterior to the hypothalamus

The answer is C. The thymus is located anteriorly in the upper part of the chest behind the sternum in between the lungs.

The neurologist is conducting a Tensilon test (Edrophonium) at the bedside of a patient who is experiencing unexplained muscle weakness, double vision, difficulty breathing, and ptosis. Which findings after the administration of Edrophonium would represent the patient has myasthenia gravis? A. The patient experiences worsening of the muscle weakness. B. The patient experiences wheezing along with facial flushing. C. The patient reports a tingling sensation in the eyelids and sudden ringing in the ears. D. The patient experiences improved muscle strength.

The answer is D. During a Tensilon test Edrophonium is administered. This medication prevents the breakdown of acetylcholine, which will allow more of the neurotransmitter acetylcholine to be present at the neuromuscular junction....hence IMPROVING muscle strength IF myasthenia gravis is present. Therefore, if a patient with MG is given this medication they will have improved muscle strength.

Which patient below is MOST at risk for developing a cholinergic crisis? A. A patient with myasthenia gravis is who is not receiving sufficient amounts of their anticholinesterase medication. B. A patient with myasthenia gravis who reports not taking the medication Pyridostigmine for 2 weeks. C. A patient with myasthenia gravis who is experiencing a respiratory infection and recently had left hip surgery. D. A patient with myasthenia gravis who reports taking too much of their anticholinesterase medication.

The answer is D. Remember patients who experience a cholinergic crisis are most likely to because they've received too much of their anticholinesterase medications (example Pyridostigmine). However, on the other hand, patients who have received insufficient amount of their anticholinesterase medication or have experienced an illness/stress/surgery are most likely to experience a myasthenia crisis. Both conditions will lead to muscle weakness and respiratory failure but from different causes, which is why a Tensilon test is used to help differentiate between the two conditions.

True or False: Parkinson's Disease most commonly affects patients in young adulthood, and there is currently no cure for the disease.

The answer is FALSE. Parkinson's Disease most commonly affects patients in OLDER adulthood (60 or older), and there is currently no cure for the disease.

A patient is suspected of having multiple sclerosis. The neurologist orders various test. The patient's MRI results are back and show lesions on the cerebellum and optic nerve. What signs and symptoms below would correlate with this MRI finding in a patient with multiple sclerosis? A. Blurry vision B. Pain when moving eyes C. Dysarthria D. Balance and coordination issues E. "Pill rolling" of fingers and hands G. Heat intolerance H. Dark spots in vision I. Ptosis

The answers are A, B C, D, and H. If lesions are present on the optic nerves, optic neuritis can occurs which can lead to blurry vision, pain when moving the eyes, and dark spots in the vision. If cerebellar lesions are found, this can affect movement, speech, and some cognitive abilities. This would present as dysarthria (issues articulating words), and balance/coordination issues. "Pill rolling" of the fingers and hands is found in Parkinson's disease. Ptosis is common in myasthenia gravis, and heat intolerance in thyroid issues.

A patient is taking Rasagiline "Azilect" for treatment of Parkinson's Disease. What foods do the patient want to limit in their diet? Select all that apply: A. Liver B. Aged Cheese C. Sweetbread D. Beer E. Fermented foods F. Shellfish

The answers are B, D, E. Rasagiline "Azilect" is a MAO Inhibitor Type B (Monoamine Oxidase Inhibitor). The patient should avoid foods high in tyramine which can cause a hypertensive crisis. This includes: aged cheese, smoked/cured meats, fermented food, beer.

Which of the following statements regarding hand tremors in multiple sclerosis is accurate? 1. Tremors increase as the client's blood glucose rises 2. Tremors increase when anxiety increases 3. Tremors increase when the client is asleep 4. Tremors will increase as activity increases

Tremors will increase as activity increases (intentional tremor)

A patient is jaundiced, and her stools are clay colored. This is most likely related to a. decreased bile flow into the intestine. b. increased production of urobilinogen. c. increased bile and bilirubin in the blood. d. increased production of cholecystokinin.

a.

A client presents to the emergency department, complaining of cramping pain in the left lower quadrant, fatigue, bloating, malaise, and a temperature of 100.3 F. Which condition does the client most likely possess? a. Cholecystitis b. Acute Pancreatitis c. Appendicitis d. Diverticulitis

answer: Diverticulitis Cholecystitis-RUQ Acute Pancreatitis- LUQ Appendicitis-RLQ

A client with hepatitis A is being discharged. Which symptoms would the nurse instruct the client to contact the primary health care provider for? a) Anorexia b) Clay-colored stools c) Fatigue d) Yellow urine

b) Clay-colored stools indicates hepatic obstruction

2. A 65-yr-old woman was just diagnosed with Parkinson's disease. The priority nursing intervention is a. searching the Internet for educational videos. b. helping the caregiver explore respite care options. c. promoting physical exercise and a well-balanced diet. d. teaching about the benefits and risks of ablation surgery.

c. Rationale: Promoting physical exercise and a well-balanced diet are major concerns of nursing care for patients newly diagnosed with Parkinson's disease. Diet is of major concern as malnutrition and constipation can result from poor nutrition. Exercise can limit the effects of decreased mobility, such as muscle atrophy, contractures, and constipation. Helping the patient and caregiver explore respite care options may be appropriate later in the course of PD.

Which is an expected finding for a client admitted with cholelithiasis and obstructive jaundice? Coffee-ground emesis Straw-colored urine Clay-colored stools Painful urination

clay-colored stools The feces, no longer colored with bile pigments, are grayish, like putty, or clay-colored. Obstructive Jaundice: Obstructive jaundice is a specific type of jaundice, where symptoms develop due to a narrowed or blocked bile duct or pancreatic duct, preventing the normal drainage of bile from the bloodstream into the intestines. Causes: • Cirrhosis, hepatitis, liver cancer • Common bile duct obstruction from stone(s), biliary strictures, pancreatic cancer, sclerosing cholangitis Description: • Results from decreased or obstructed flow of bile through liver or biliary duct system • Obstruction may occur in intrahepatic or extrahepatic bile ducts • Intrahepatic obstructions are due to swelling or fibrosis of the liver's canaliculi and bile ducts Manifestations: *pruritis *light stool *dark urine *jaundice

What is the cause of the skin color change in a client with hepatitis? a) There are decreased prothrombin levels and this leads to intradermal bleeding. b) Simulation of the liver produces excess bile pigments c) There are increased destruction of red blood cells during the acute phase of the disease d) The liver has lost the ability to remove normal amounts of bilirubin from the blood

d

What type of hepatitis is spread through contaminated food and water or by fecal contamination? a) Hepatitis A and B b) Hepatitis D and E c) Hepatitis C and D d) Hepatitis A and E

d Hepatitis A is spread by either direct contact with an infected person's feces or by indirect fecal contamination of food or water. There is a vaccine to prevent against infection. Can live outside of the body for at least seven days. During that time, the virus is still capable of causing infection . Hepatitis E virus is found in underdeveloped areas of the world and is spread by the fecal-oral route. Hepatitis E causes acute hepatitis, which usually goes away on its own. It can be more dangerous in pregnant women who are at an increased risk of liver failure and death. Hepatitis E does not cause chronic infection.

A client is admitted to the progressive care unit with a diagnosis of cirrhosis. What assessment finding would be expected? A. Evidence of vitiligo on the skin B. Evidence of hirsutism C. Appearance of melanosis on the skin D. Small, scattered ecchymoses on the bilateral arms and legs

d. Manifestations of coagulation problems (bleeding tendencies) include epistaxis, purpura, petechiae, easy bruising, gingival bleeding, and heavy menstrual bleeding. Spider angiomas (telangiectasia or spider nevi) are small, dilated blood vessels with a bright red center point and spiderlike branches. They occur on the nose, cheeks, upper trunk, neck, and shoulders. Palmar erythema (a red area that blanches with pressure) occurs on the palms of the hands. Both lesions are due to an increase in circulating estrogen due to the damaged liver's inability to metabolize steroid hormones.

Which statement by a student nurse is correct regarding interventions to reduce the pancreatic and gastric secretions of a client with pancreatitis? "We should keep the client in a semi-Fowler position." "We should talk to the provider about an order for PRN morphine." "The client needs to drink more clear liquids to prevent dehydration." "I will administer the prescribed anticholinergic medication."

"I will administer the prescribed anticholinergic medication." Rationale: Anticholinergic drugs block the neural impulses that stimulate pancreatic and gastric secretions; they inhibit the action of acetylcholine at postganglionic cholinergic nerve fibers. Oral fluids stimulate pancreatic secretion and are contraindicated. Morphine sulfate is an analgesic and therefore does not decrease gastric secretions; in the past morphine sulfate was contraindicated for pain control with pancreatitis because it can precipitate spasms of the smooth musculature of the pancreatic ducts and the sphincter of Oddi. However, recent research indicates that it is the drug of choice over meperidine hydrochloride because the metabolites of meperidine hydrochloride can cause central nervous system irritation and seizures. The semi-Fowler position decreases pressure against the diaphragm; it will not decrease pancreatic secretions. https://quizlet.com/494611154/4510-pancreatitischolecystitis-flash-cards/

A client is receiving discharge instructions after treatment for cholelithiasis, with a plan to remove the gallbladder in two weeks. Which instruction would be necessary to include regarding diet? A. "You need to eat more protein for tissue healing and improved energy." B. "Eat low cholesterol foods to avoid further buildup of your gallstones." C. "Eat soft-textured foods that will be easy on your stomach." D. "Make sure you reduce your fat intake because this stimulates the cholecystokinin mechanism for bile release."

"Make sure you reduce your fat intake because this stimulates the cholecystokinin mechanism for bile release."

A client is diagnosed with chronic Hepatitis B and asks about a cure. Which response will be the most appropriate? "There are medications that can lower the viral load and reduce liver inflammation." "Sedatives would help you not to be anxious about this." "There is a medication called immune serum globulin that can cure you." "A high dose of vitamin C would help hasten recover."

"There are medications that can lower the viral load and reduce liver inflammation." Drugs are available to help reduce the viral load (antivirals), including lamivudine, rib a virgin, and adefovir dipivoxil. Although sedatives can be given to help relax, sedatives are given only as needed and do not react the hepatitis. The response "We can give you immune serum globulin" is used only during the incubation period. Vitamins are used as adjective therapy and will not eliminate the hepatitis.

The nurse is aware of potential complications related to cirrhosis. Which interventions would be included in a safe plan of care? (Select all that apply.) 1. Provide a high-protein, low-carbohydrate diet. 2. Tell the patient to use soft-bristle toothbrush and electric razor. 3. Teach the patient to avoid vigorous blowing of nose and coughing. 4. Apply gentle pressure for the shortest possible time after venipuncture. 5. Use the smallest gauge needle possible when giving injections or drawing blood. 6. Teach the patient to avoid aspirin and nonsteroidal antiinflammatory (NSAIDs).

2, 3, 5, 6 Using the smallest gauge needle for injections, using a soft bristle toothbrush and an electric razor will minimize the risk of bleeding into the tissues. Avoiding straining, nose blowing, and coughing will reduce the risk of hemorrhage at these sites. The nurse should apply gentle but prolonged pressure to venipuncture sites to minimize the risk of bleeding. Aspirin and NSAIDs should not be used in patients with liver disease because they interfere with platelet aggregation, thus increasing the risk for bleeding. A low-salt, low-protein, high-carbohydrate diet may be recommended.

The nurse is caring for a patient with acute pancreatitis resulting from gallstones. Which clinical manifestation would the nurse expect? (Select all that apply.) 1. Hematochezia 2. Nausea and vomiting 3. Hyperactive bowel sounds 4. Left upper abdominal pain 5. Ascites and peripheral edema 6. Temperature 99.3° F (37.4° C)

2, 4, 6 Abdominal pain (usually in the left upper quadrant) is the predominant manifestation of acute pancreatitis. Other manifestations include nausea and vomiting, low-grade fever, leukocytosis, hypotension, tachycardia, and jaundice. Abdominal tenderness with muscle guarding is common. Bowel sounds may be decreased or absent. Ileus may occur and causes marked abdominal distention. Areas of cyanosis or greenish to yellow-brown discoloration of the abdominal wall may occur. Other areas of ecchymoses are the flanks (Grey Turner's spots or sign, a bluish flank discoloration) and the periumbilical area (Cullen's sign, a bluish periumbilical discoloration).

When providing discharge teaching for a patient after a laparoscopic cholecystectomy, what information should the nurse include? 1. Do not return to work or normal activities for 3 weeks. 2. A low-fat diet may be better tolerated for several weeks. 3. Bile-colored drainage will probably drain from the incision. 4. Keep the bandages on and the puncture site dry until it heals.

2. Although the usual diet can be resumed, a low-fat diet is usually better tolerated for several weeks after surgery. Normal activities can be gradually resumed as the patient tolerates. Bile-colored drainage or pus, redness, swelling, severe pain, and fever may all indicate infection. The bandage may be removed the day after surgery, and the patient can shower.

A client with a history of alcoholism is admitted for acute pancreatitis. How does the client's alcohol use relate to the diagnosis of pancreatitis? 1. It stimulates the pancrease to secrete more insulin that it can immediately produce 2. Chronic alcohol use causes enzyme secretion and puts increased pressure on the pancreatic duct, causing a backflow of enzymes into the pancreas 3. It promotes the formation of gallstones in the cystic duct 4. It alters the composition of enzymes so that they begin to damage the pancreas

2. Chronic alcohol use causes enzyme secretion and puts increased pressure on the pancreatic duct, causing a backflow of enzymes into the pancreas Rationale: Alcohol stimulates pancreatic enzyme secretion and an increase in pressure in the pancreatic duct. The backflow of enzymes into the pancreatic interstitial spaces results in partial digestion and inflammation of the pancreatic tissue. Although blockage of the bile duct with calculi may precipitate pancreatitis, this is not associated with alcohol. Alcohol does not deplete insulin stores; the demand for insulin is unrelated to pancreatitis. Although the volume of secretions increases, the composition remains unchanged. https://quizlet.com/494611154/4510-pancreatitischolecystitis-flash-cards/

***A patient with sudden pain in the left upper quadrant radiating to the back and vomiting was diagnosed with acute pancreatitis. Which intervention should the nurse include in the patient's plan of care? 1. Immediately start enteral feeding to prevent malnutrition. 2. Insert an NG and maintain NPO status to allow pancreas to rest. 3. Initiate early prophylactic antibiotic therapy to prevent infection. 4. Administer acetaminophen (Tylenol) every 4 hours for pain relief.

2. Initial treatment with acute pancreatitis will include an NG tube if there is vomiting and being NPO to decrease pancreatic enzyme stimulation and allow the pancreas to rest and heal. Fluid will be administered to treat or prevent shock. The pain will be treated with IV morphine because of the NPO status. Enteral feedings will only be used for the patient with severe acute pancreatitis in whom oral intake is not resumed. Antibiotic therapy is only needed with acute necrotizing pancreatitis and signs of infection.

A patient with chronic hepatitis B is being discharged with pain medication after knee surgery. Which medication order should the nurse question? 1. Tramadol 2. Hydromorphone (Dilaudid) 3. Hydrocodone with acetaminophen 4. Oxycodone with aspirin (Percodan)

3. The analgesic with acetaminophen should be questioned because this patient has chronic hepatitis B and is likely to have impaired liver function. Acetaminophen is not suitable for this patient because it is converted to a toxic metabolite in the liver after absorption, increasing the risk of hepatocellular damage.

A patient with cirrhosis has increased abdominal girth from ascites. Which statements describe the pathophysiology of ascites? (Select all that apply.) 1. Hepatocytes are unable to convert ammonia to urea. 2. Osmoreceptors in the hypothalamus stimulate thirst. 3. An enlarged spleen removes blood cells from the circulation. 4. Portal hypertension causes leaking of protein and water into the peritoneal cavity. 5. Aldosterone is released to stabilize intravascular volume by saving salt and water. 6. Inability of the liver to synthesize albumin reducing intravascular oncotic pressure.

4, 5, 6. Ascites related to cirrhosis is caused by decreased colloid oncotic pressure. The liver does not produce albumin that holds fluid in the vascular space, so fluid shifts into interstitial and third spaces. Portal hypertension causes back pressure in the vessels, shifting protein and fluids into the peritoneal cavity. Decreased intravascular volume stimulates the release of aldosterone, which increases sodium and fluid retention. Oral intake of fluids and removal of blood cells by the spleen do not directly contribute to ascites.

A patient with cholelithiasis is being prepared for surgery. Which patient assessment represents a contraindication for a cholecystectomy? 1. Low-grade fever of 100° F and dehydration 2. Abscess in the right upper quadrant of the abdomen 3. Multiple obstructions in the cystic and common bile duct 4. Activated partial thromboplastin time (aPTT) of 54 seconds

4. An aPTT of 54 seconds is above normal and indicates insufficient clotting ability. If the patient had surgery, significant bleeding complications postoperatively are very likely. Fluids can be given to eliminate the dehydration. The abscess can be assessed during surgery, and the obstructions in the cystic and common bile duct would be relieved with the cholecystectomy.

A patient with a history of lung cancer and hepatitis C has developed liver failure and is considering liver transplantation. After a comprehensive evaluation, which finding may be a contraindication for liver transplantation? 1. History of hypothyroidism 2. Stopped smoking cigarettes 3. Well-controlled type 1 diabetes 4. Chest x-ray shows a new lung cancer lesion

4. Contraindications for liver transplant include severe extrahepatic disease, advanced hepatocellular carcinoma or other cancer, ongoing drug or alcohol use, and the inability to comprehend or comply with the rigorous posttransplant course.

The health care provider orders lactulose for a patient with hepatic encephalopathy. Which finding indicates the medication has been effective? 1. Relief of constipation 2. Relief of abdominal pain 3. Decreased liver enzymes 4. Decreased ammonia levels

4. Hepatic encephalopathy is a complication of liver disease and is associated with elevated serum ammonia levels. Lactulose traps ammonia in the intestinal tract. Its laxative effect then expels the ammonia from the colon, resulting in decreased serum ammonia levels and correction of hepatic encephalopathy. An additional finding may be an improvement in level of consciousness.

***The family of a patient newly diagnosed with hepatitis A asks the nurse what they can do to prevent becoming ill. Which response by the nurse is most appropriate? 1. "You will need to be tested first; then treatment can be determined." 2. "The hepatitis vaccine will provide immunity from this and future exposures." 3. "There is nothing you can do since the patient was infectious before admission." 4. "An immunoglobulin injection will be given to prevent infection or limit symptoms."

4. Immunoglobulin provides temporary (1 to 2 months) passive immunity and is effective for preventing hepatitis A if given within 2 weeks after exposure. It may not prevent infection in all persons, but it will at least modify the illness to a subclinical infection. The hepatitis vaccine is only used for preexposure prophylaxis.

When teaching the patient with acute hepatitis C (HCV), which statement demonstrates understanding of the disease process? 1. "I will use care when kissing my wife to prevent giving it to her." 2. "I will need to take adefovir (Hepsera) to prevent chronic HCV." 3. "Now that I have had HCV, I will have immunity and not get it again." 4. "I will need to be monitored for chronic HCV and other liver problems."

4. Many patients who acquire HCV develop chronic infection, which may lead to cirrhosis or liver cancer. HCV is not transmitted via saliva but by blood exposures such as sharing needles and high-risk sexual activity. The treatment for acute viral hepatitis focuses on resting the body and adequate nutrition for liver regeneration. Adefovir (Hepsera) is taken for severe hepatitis B (HBV) with liver failure. HCV is treated with oral direct-acting antivirals (DAAs). Immunity with HCV does not occur as it does with HAV and HBV, so the patient may be reinfected with another type of HCV.

The patient with suspected pancreatic cancer is having many diagnostic studies done. Which test can be used to establish the diagnosis of pancreatic cancer and for monitoring the response to treatment? 1. Spiral CT scan 2. A PET/CT scan 3. Abdominal ultrasound 4. Cancer-associated antigen 19-9

4. The cancer-associated antigen 19-9 (CA 19-9) is the tumor marker used for the diagnosis of pancreatic cancer and monitoring the response to treatment. Although a spiral CT scan may be the initial study done and provides information on metastasis and vascular involvement, this test and the positron emission tomography (PET)/CT scan or abdominal ultrasonography does not provide additional information.

A patient with hepatitis A is in the acute phase. The nurse plans care for the patient based on the knowledge that a. itching is a common problem with jaundice in this phase. b. the patient is most likely to transmit the disease during this phase. c. gastrointestinal symptoms are not as severe in hepatitis A as they are in hepatitis B. d. extrahepatic manifestations of glomerulonephritis and polyarteritis are common in this phase.

A, Rationale: The acute phase of jaundice may be icteric or anicteric. Jaundice results when bilirubin diffuses into the tissues. Pruritus sometimes accompanies jaundice. Pruritus is the result of an accumulation of bile salts beneath the skin.

Nursing management of the patient with acute pancreatitis includes (select all that apply) a. administering pain medication. b. checking for signs of hypocalcemia. c. providing a diet low in carbohydrates. d. giving insulin based on a sliding scale. e. monitoring for infection, particularly respiratory tract infection.

A, b, e. Rationale: During the acute phase, it is important to provide pain relief and monitor vital signs. Hypotension, fever, and tachypnea may compromise hemodynamic stability. Give fluids and monitor the response to therapy. Closely monitor fluid and electrolyte balances. Vomiting and gastric suction may result in decreased chloride, sodium, and potassium levels. Because hypocalcemia can occur, observe for symptoms of tetany, such as jerking, irritability, and muscular twitching. Numbness or tingling around the lips and in the fingers is an early sign of hypocalcemia. Assess the patient for Chvostek's sign or Trousseau's sign. Observe for fever and other manifestations of infection. Respiratory infections are common because the retroperitoneal fluid raises the diaphragm, causing the patient to take shallow, guarded abdominal breaths.

Which nursing intervention is the most appropriate for a client admitted after a laparoscopic cholecystectomy? a. Keep the client on strict bedrest for 12 hours b. Place the client on a morphine PCA with a basal rate of 2mg/hr. c. Ambulate the client when the client awakens and is alert d. Keep the client NPO for 24 hours

Ambulate the client when the client awakens and is alert

Which laboratory test is necessary to determine if a client with acute pancreatitis is responding to treatment?Platelet count Red blood cell count Erythrocyte sedimentation rate Amylase level

Amylase level Rationale: In 90% of clients with acute pancreatitis, the amylase level is elevated up to three times over baseline; serum amylase usually returns to expected adult levels within three days after treatment begins. The platelet count is not an indicator of the response to treatment for pancreatitis; platelets are important in the control of bleeding. The red blood cell count is unchanged in acute pancreatitis, unless hemorrhage is present. The erythrocyte sedimentation rate is not an indicator of a response to treatment for pancreatitis. https://quizlet.com/494611154/4510-pancreatitischolecystitis-flash-cards/

The child is diagnosed with hepatitis A. The clients parent expresses concern of the other members of the family may get hepatitis because they all share the same bathroom. What is the nurses best reply? A. "I suggest that you buy come out exclusively for your child use." B. "Your child me use the bathroom but you need to use disposable toilet covers." C. "You will need to clean the bathroom from top to bottom every time a family member uses it." D. "All family members including your child need to wash their hands after using the bathroom."

Answer: D. "All family members including your child need to wash their hands after using the bathroom." Hepatitis A is spread via the fecal-oral route; transmission is prevented by proper hand washing. Buying a commode exclusively for the child's use is unnecessary; cleaning the toilet and washing the hands should control the transmission of microorganisms. It is not feasible to clean "from top to bottom" each time the bathroom is use. The use of disposable toilet cover is inadequate to prevent the spread of microorganisms if the bathroom used by the child also is used by other.

A client is about to have a blood transfusion and asks the nurse which type of hepatitis is most frequently transmitted through food. Which type of hepatitis should the nurse teach the client about being most associated with food? A. A B. B C. C D. D

Answer: A. A Hepatitis A, also known as infectious hepatitis, is cause by an RNA virus that is transmitted via the fecal-oral route and is most frequently transmitted through food. Hepatitis B is transmitted parenterally, sexually, and by direct contact with infected body secretion. Hepatitis C is cause by an RNA virus that is transmitted parenterally. Hepatitis D is a complication of hepatitis B. Hand washing by all family members must be part of the plan to prevent the spread of hepatitis to other family members.

A client has developed hepatitis A after eating contaminated oysters. The nurse assesses the client for which expected assessment finding? A. Malaise B. Dark stools C. Weight gain D. Left upper quadrant discomfort

Answer: A. Malaise Hepatitis causes GI symptoms such as anorexia, nausea, right upper quadrant discomfort and weight loss. Fatigue and malaise are common. Stools will be light or clay colored if conjugated bilirubin is unable to flow out of the liver because of inflammation or obstruction of the bile ducts.

The registered nurse determines that the new graduate understands the type(s) of hepatitis that generally develop into a chronic hepatitis infection if the graduate identifies which disease(s)? Select all that apply. A. Hepatitis A B. Hepatitis B C. Hepatitis C D. Hepatitis D E. Hepatitis E

Answer: B, C, D: Hepatitis B and C generally develop into chronic hepatitis. Hepatitis D is an incomplete virus that can become chronic and is dependent on the presence of hepatitis B to survive. Hepatitis A and E are acute, self-limiting infections that resolve over time and do not develop into chronic hepatitis.

A client with jaundice associated with hepatitis expresses concern over the change in skin color. What does the nurse explain is the cause of this color change? A. Stimulation of the liver to produce an excess quantity of bile pigments B. Inability of the liver to remove normal amounts of bilirubin from the blood C. Increased destruction of red blood cells during the acute phase of the disease. D. Decreased prothrombin levels, leading to multiple sites of intradermal bleeding

Answer: B. Inability of the liver to remove normal amounts of bilirubin from the bloodDamage to liver cells affects the ability to facilitate removal of bilirubin from the blood, with resulting deposition in the skin and sclera. With hepatitis, the liver does not secrete excess bile. Destruction of red blood cells does not increase hepatitis. Decreased prothrombin levels cause spontaneous bleeding, not jaundice.

A nurse is teaching a client about prophylactic measures that minimize the risk of contracting hepatitis B. Which actions should be included in this teaching plan? Select all that apply. A. Preventing constipation B. Screening of blood donors C. Avoiding shellfish in the diet D. Limiting hepatotoxic drug therapyE. Maintaining a monogamous sexual relationship

Answer: B. Screening of blood donors, E. Maintaining a monogamous sexual relationshipContracting hepatitis B through blood transfusion can be prevented by screening donors and testing the blood. Hepatitis B can be transmitted via contaminated body fluids such as semen, saliva, and urine. Having multiple sexual partners increases the risk. A monogamous sexual relationship with an infection-free individual eliminates the risk. Preventing constipation is not related to limiting the risk for contracting hepatitis B. Avoid shellfish in the diet limits the risk for contracting hepatitis A. Limiting hepatotoxic drug therapy does not prevent transmission of hepatitis B.

The primary health care provider prescribed contact precautions for a client with hep A. What nursing intervention are required for contact precautions? A. Private room with door closed B. Gown, mask, and gloves for all persons entering the room C. Gown and gloves when handling articles contaminated by urine or feces D. Gowns and gloves only when handling the client's soiled license, dishes, or utensils.

Answer: C. Gown and gloves when handling articles contaminated by urine or fecesHepatitis A is transmitted via the fecal-oral route; contact precautions must be used when there are articles that potential fecal or urine contamination. Neither a private room nor closed door is required; these are necessary only for respiratory (airborne) precautions. Hepatitis A is not transmitted via the airborne route and therefore a mask is not necessary; a gown and gloves are required only one handling articles that may be contaminated. Wearing gowns and gloves only one handling the client soiled linen, dishes, or utensils is too limited; a gown and gloves also should be worn when handling other fecally contaminated articles, such as a bedpan or rectal thermometer.

The client with hep B asks the nurse, "Are there any medication to help me get rid of this problem?" Which is the best response by the nurse? A. "Sedatives can be given to help you relax." B. "We can give you immune serum globulin." C. "Vitamin supplements are frequently helpful and hasten recovery." D. "There are medication to help reduce viral load and liver inflammation."

Answer: D. "There are medication to help reduce viral load and liver inflammation."Drugs are available to help reduce the viral load (antivirals), including lamivudine, rib a virgin, and adefovir dipivoxil. Although sedatives can be given to help relax, sedatives are given only as needed and do not react the hepatitis. The response "We can give you immune serum globulin" is used only during the incubation period. Vitamins are used as adjective therapy and will not eliminate the hepatitis.

The nurse caring for a patient with suspected acute cholecystitis would anticipate (select all that apply) a. ordering a low-sodium diet. b. administration of IV fluids. c. monitoring of liver function tests. d. administration of antiemetics for patients with nausea. e. insertion of an indwelling catheter to monitor urinary output.

B, c, d. Rationale: Priorities for patients with acute cholecystitis include pain management and control of symptoms, such as nausea. Maintaining fluid and electrolyte balance are important. IV access and fluids can help address this issue. Monitoring laboratory values, in particular liver function tests, can provide information on the severity of the acute episode.

A patient with acute hepatitis B is being discharged. The discharge teaching plan should include instructions to a. avoid alcohol for the first 3 weeks. b. use a condom during sexual intercourse. c. have family members get an injection of immunoglobulin. d. follow a low-protein, moderate-carbohydrate, moderate-fat diet.

B. Rationale: Hepatitis B virus may be transmitted by mucosal exposure to infected blood, blood products, or other body fluids (e.g., semen, vaginal secretions, saliva). Hepatitis B is a sexually transmitted disease that is acquired through unprotected sex with an infected person. Condom use should be taught to patients to prevent transmission of hepatitis B.

A client is admitted to the progressive care unit with a diagnosis of cholelithiasis and obstructive jaundice. Which is an expected finding? Ecchymosis Crackles in bilateral lung bases Constipation Straw-colored urine

Ecchymosis

A client is scheduled for an abdominal cholecystectomy. Which intervention would be a priority in the immediate post-operative period? Education on restrictions of food and fluids Encouraging coughing and deep breathing Education on the signs and symptoms of infection Isometric exercises to perform while in the hospital.

Encouraging coughing and deep breathing Rationale: The client who has a cholecystectomy will have difficulty taking deep breaths and coughing because of the location of the surgical incision. Therefore it is important to instruct the client preoperatively to improve compliance with the procedure in the early postop period. Although ambulation, antiembolism stockings, and maintaining a nasogastric tube, if ordered, are important postoperative procedures, maintaining the airway and preventing further pulmonary problems is the priority. https://quizlet.com/494611154/4510-pancreatitischolecystitis-flash-cards/

Which dietary considerations will have to be made for a client who had surgery for pancreatic cancer, and has less functioning pancreatic tissue? Consider the functions of the pancreas when determining your answer. Alcohol and caffeine Fluids and electrolytes Vitamins and minerals Fats and carbohydrates

Fats and carbs Less fat, more carbs

Which type of hepatitis is associated with tattoos from non-professional sources? a) Hepatitis D b) Hepatitis E c) Hepatitis C d) Hepatitis A

Hep C

-What clinical manifestation would the nurse expect in a client newly admitted with severe acute pancreatitis? Hypotension Hypoglycemia Hypertension Polyuria

Hypotension

Which clinical finding is expected for a client admitted with acute cholecystitis? Coffee-ground emesis Dull, throbbing pain with meals Tarry stools Intolerance to foods high in fat

Intolerance to foods high in fat

In this surgical procedure, the pancreatic duct is anastomosed to the jejunum- allowing pancreatic enzymes to bypass the common bile duct and gallbladder? _____________________

Roux-en-Y

T/F: Autodigestion is the most common pathogenic mechanism in acute pancreatitis, in which pancreatic cells are injured or pancreatic enzymes are activated in the pancreas

TRUE The most common pathogenic mechanism in acute pancreatitis is autodigestion of the pancreas. The causative factors injure pancreatic cells or activate the pancreatic enzymes in the pancreas rather than in the intestine. This may be due to reflux of bile acids into the pancreatic ducts through an open or distended sphincter of Oddi. This reflux may be caused by blockage created by gallstones. Obstruction of pancreatic ducts results in pancreatic ischemia

***A patient has lab work drawn and it shows a positive HBsAg. What education will you provide to the patient? A. Avoid sexual intercourse or intimacy such as kissing until blood work is negative. B. The patient is now recovered from a previous Hepatitis B infection and is now immune. C. The patient is not a candidate from antiviral or interferon medications. D. The patient is less likely to develop a chronic infection.

The answer is A. A positive HBsAg (hepatitis B surface antigen) indicates an active Hepatitis B infection. Therefore, the patient should avoid sexual intercourse and other forms of intimacy until their HBsAg is negative.

A patient was exposed to Hepatitis B recently. Postexposure precautions include vaccination and administration of HBIg (Hepatitis B Immune globulin). HBIg needs to be given as soon as possible, preferably ___________ after exposure to be effective. A. 2 weeks B. 24 hours C. 1 month D. 7 days

The answer is B. HBIg should be given 24 hours after exposure to maximum effectiveness of temporary immunity against Hepatitis B. It would be given within 12 hours after birth to an infant born to a mother who has Hepatitis B.

***A 36-year-old patient's lab work show anti-HAV and IgG present in the blood. As the nurse you would interpret this blood work as? A. The patient has an active infection of Hepatitis A. B. The patient has recovered from a previous Hepatitis A infection and is now immune to it. C. The patient is in the preicetric phase of viral Hepatitis. D. The patient is in the icteric phase of viral Hepatitis.

The answer is B. When a patient has anti-HAV (antibodies of the Hepatitis A virus) and IgG, this means the patient HAD a past infection of Hepatitis A but it is now gone, and the patient is immune to Hepatitis A now. If the patient had anti-HAV and IgM, this means the patient has an active infection of Hepatitis A.

What is the BEST preventive measure to take to help prevent ALL types of viral Hepatitis? A. Vaccination B. Proper disposal of needles C. Hand hygiene D. Blood and organ donation screening

The answer is C. Hand hygiene can help prevent all types of viral hepatitis. However, not all types of viral Hepatitis have a vaccine available or are spread through needle sticks or blood/organs donations. Remember Hepatitis A and E are spread only via fecal-oral routes.

The physician writes an order for the administration of Lactulose. What lab result indicates this medication was successful? A. Bilirubin <1 mg/dL B. ALT 8 U/L C. Ammonia 16 mcg/dL D. AST 10 U/L

The answer is C. Lactulose is ordered to decrease a high ammonia level. It will cause excretion of ammonia via the stool. A normal ammonia level would indicate the medication was successful (normal ammonia level 15-45 mcg/dL).

A patient with Hepatitis A asks you about the treatment options for this condition. Your response is? A. Antiviral medications B. Interferon C. Supportive care D. Hepatitis A vaccine

The answer is C. There is no current treatment for Hepatitis A but supportive care and rest. Treatments for the other types of Hepatitis such as B, C, and D include antiviral or interferon (mainly the chronic cases) along with rest.

A patient is prescribed Peginterferon alfa-2a. The nurse will prepare to administer this medication what route? A. Oral B. Intramuscular C. Subcutaneous D. Intravenous

The answer is C. This medication is administered subq.

***A patient has completed the Hepatitis B vaccine series. What blood result below would demonstrate the vaccine series was successful at providing immunity to Hepatitis B? A. Positive IgG B. Positive HBsAg C. Positive IgM D. Positive anti-HBs

The answer is D. A positive anti-HBs (Hepatitis B surface antibody) indicates either a past infection of Hepatitis B that is now cleared and the patient is immune, OR that the vaccine has been successful at providing immunity. A positive HBsAg (Hepatitis B surface antigen) indicates an active infection.

A patient is diagnosed with Hepatitis A. The patient asks how a person can become infected with this condition. You know the most common route of transmission is? A. Blood B. Percutaneous C. Mucosal D. Fecal-oral

The answer is D. Hepatitis A is most commonly transmitted via the fecal-oral route.

16. What is the MOST common transmission route of Hepatitis C? A. Blood transfusion B. Sharps injury C. Long-term dialysis D. IV drug use

The answer is D. IV drug use is the MOST common transmission route of Hepatitis C.

A 25-year-old patient was exposed to the Hepatitis A virus at a local restaurant one week ago. What education is important to provide to this patient? A. Inform the patient to notify the physician when signs and symptoms of viral Hepatitis start to appear. B. Reassure the patient the chance of acquiring the virus is very low. C. Inform the patient it is very important to obtain the Hepatitis A vaccine immediately to prevent infection. D. Inform the patient to promptly go to the local health department to receive immune globulin.

The answer is D. Since the patient was exposed to Hepatitis A, the patient would need to take preventive measures to prevent infection because infection is possible. The patient should not wait until signs and symptoms appear because the patient can be contagious 2 weeks BEFORE signs and symptoms appear. The vaccine would not prevent Hepatitis A from this exposure, but from possible future exposures because it takes the vaccine 30 days to start working. The best answer is option D. The patient would need to receive immune globulin to provide temporary immunity within 2 weeks of exposure.

Select all the types of viral Hepatitis that have preventive vaccines available in the United States? A. Hepatitis A B. Hepatitis B C. Hepatitis C D. Hepatitis D E. Hepatitis E

The answers are A and B. Currently there is only a vaccine for Hepatitis A and B in the U.S.

A patient is diagnosed with Hepatitis D. What statement is true about this type of viral Hepatitis? Select all that apply: A. The patient will also have the Hepatitis B virus. B. Hepatitis D is most common in Southern and Eastern Europe, Mediterranean, and Middle East. C. Prevention of Hepatitis D includes handwashing and the Hepatitis D vaccine. D. Hepatitis D is most commonly transmitted via the fecal-oral route.

The answers are A and B. These are true statements about Hepatitis D. Prevention for Hepatitis D includes handwashing and the Hepatitis B vaccine (since it occurs only with the Hepatitis B virus). It is transmitted via blood.

Which statements are INCORRECT regarding the anatomy and physiology of the liver? Select all that apply: A. The liver has 3 lobes and 8 segments. B. The liver produces bile which is released into the small intestine to help digest fats. C. The liver turns urea, a by-product of protein breakdown, into ammonia. D. The liver plays an important role in the coagulation process.

The answers are A and C. The liver has 2 lobes (not 3), and the liver turns ammonia (NOT urea), which is a by-product of protein breakdown, into ammonia. All the other statements are true about liver's anatomy and physiology.

You're providing an in-service on viral hepatitis to a group of healthcare workers. You are teaching them about the types of viral hepatitis that can turn into chronic infections. Which types are known to cause ACUTE infections ONLY? Select all that apply: A. Hepatitis A B. Hepatitis B C. Hepatitis C D. Hepatitis D E. Hepatitis E

The answers are A and E. Only Hepatitis A and E cause ACUTE infections...not chronic. Hepatitis B, C, and D can cause both acute and chronic infections.

Select all the signs and symptoms associated with Hepatitis? A. Arthralgia B. Bilirubin 1 mg/dL C. Ammonia 15 mcg/dL D. Dark urine E. Vision changes F. Yellowing of the sclera G. Fever H. Loss of appetite

The answers are A, D, F, G, and H. The bilirubin and ammonia levels are normal in these options, but they would be abnormal in Hepatitis. A normal bilirubin is 1 or less, and a normal ammonia is 15-45 mcg/dL.

During the posticteric phase of Hepatitis the nurse would expect to find? Select all that apply: A. Increased ALT and AST levels along with an increased bilirubin level B. Decreased liver enzymes and bilirubin level C. Flu-like symptoms D. Resolved jaundice and dark urine

The answers are B and D. Posticteric (convalescent) Phase: jaundice and dark urine start to subside and stool returns to normal brown color, liver enzymes and bilirubin decrease to normal

***Select all the ways a person can become infected with Hepatitis B: A. Contaminated food/water B. During the birth process C. IV drug use D. Undercooked pork or wild game E. Hemodialysis F. Sexual intercourse

The answers are B, C, E, and F. Hepatitis B is spread via blood and body fluids. It could be transmitted via the birthing process, IV drug use, hemodialysis, or sexual intercourse etc.

Which patients below are at risk for developing complications related to a chronic hepatitis infection, such as cirrhosis, liver cancer, and liver failure? Select all that apply: A. A 55-year-old male with Hepatitis A. B. An infant who contracted Hepatitis B at birth. C. A 32-year-old female with Hepatitis C who reports using IV drugs. D. A 50-year-old male with alcoholism and Hepatitis D. E. A 30-year-old who contracted Hepatitis E.

The answers are B, C, and D. Infants or young children who contract Hepatitis B are at a very high risk of developing chronic Hepatitis B (which is why option B is correct). Option C is correct because most cases of Hepatitis C turn into chronic cases and IV drug use increases this risk even more. Option D is correct because Hepatitis D occurs when Hepatitis B is present and constant usage of alcohol damages the liver. Therefore, the patient is at high risk of developing chronic hepatitis. Hepatitis A and E tend to only cause acute infections....not chronic.

A patient with Hepatitis has a bilirubin of 6 mg/dL. What findings would correlate with this lab result? Select all that apply: A. None because this bilirubin level is normal B. Yellowing of the skin and sclera C. Clay-colored stools D. Bluish discoloration on the flanks of the abdomen E. Dark urine F. Mental status changes

The answers are B, C, and E. This is associated with a high bilirubin level. A normal bilirubin level is 1 or less.

You're providing education to a patient with an active Hepatitis B infection. What will you include in their discharge instructions? Select all that apply: A. "Take acetaminophen as needed for pain." B. "Eat large meals that are spread out through the day." C. "Follow a diet low in fat and high in carbs." D. "Do not share toothbrushes, razors, utensils, drinking cups, or any other type of personal hygiene product." E. "Perform aerobic exercises daily to maintain strength."

The answers are C and D. The patient should NOT take acetaminophen (Tylenol) due to its effective on the liver. The patient should eat small (NOT large), but frequent meals...this may help with the nausea. The patient should rest (not perform aerobic exercises daily) because this will help with liver regeneration.

What is the preferred treatment for a chronic Hepatitis B infection? a) Hepatitis B vaccine b) Nucleoside and nucleotide analogs c) There is no treatment available, supportive care only d) Protease inhibitors

a) Nucleoside and nucleotide analogs The approved medication for the treatment of chronic HBV infection is interferon-α (IFNα) and nucleos(t)ide analogues (NAs), including lamivudine, adefovir dipivoxil, telbivudine, entecavir and tenofovir disoproxil fumarate. PPT: Drug therapy focuses on ↓ viral load , liver enzyme levels, and rate of disease progression · Prevent cirrhosis, portal hypertension, liver failure, and cancer · Nucleoside and nucleotide analogs: Rebound viral replication if drugs suddenly stopped · Interferon: Monitor for flu-like symptoms and depression

Which of the following statements by a client in the icteric phase of a hepatitis infection of unknown origin indicates a need for further education? a) "This infection better not interfere with my sex life." b) "I should eat small, frequent meals to aid in digestion." c) "I'll keep my bath towels separate from my family." d) "I'll make sure that I don't share my fork and spoon with my family."

a. CORRECT: avoid sexual contact until serologic indicators return to normal

In planning care for a patient with metastatic liver cancer, the nurse should include interventions that a. focus primarily on symptomatic and comfort measures. b. reassure the patient that chemotherapy offers a good prognosis. c. promote the patient's confidence that surgical excision of the tumor will be successful. d. provide information needed for the patient to make decisions about liver transplantation.

a. Rationale: Nursing intervention for a patient with liver cancer focuses on keeping the patient as comfortable as possible. The prognosis for patients with liver cancer is poor. The cancer grows rapidly, and death may occur within 4 to 7 months because of hepatic encephalopathy or massive blood loss from gastrointestinal (GI) bleeding.

Which education about gamma globulin should be included to a family of a client with hepatitis A that presents with fatigue and jaundice? a) "Gamma globulin isn't necessary as long as you follow droplet precautions for 2 weeks." b) "Your family member is not contagious so no gamma globulin is needed." c) "Talk to your healthcare provider as soon as you can about obtaining gamma globulin injections." d) "Gamma globulin injections provide passive immunity for hepatitis B, not hepatitis A."

a) "Talk to your healthcare provider as soon as you can about obtaining gamma globulin injections." Post-exposure prophylaxis for HAV& HBV: vaccine and immune globulin (IG). Immunoglobulin provides temporary (1 to 2 months) passive immunity and is effective for preventing hepatitis A if given within 2 weeks after exposure. It may not prevent infection in all persons, but it will at least modify the illness to a subclinical infection. The hepatitis vaccine is only used for preexposure prophylaxis.

An infant's mother has a positive HBsAG test during pregnancy. Which course of treatment would be expected for the newborn? a) Hepatitis B immune globulin is given within a week after obtaining the birth parents' laboratory results b) Hepatitis B immune globulin 0.5ml is given along with the first dose of Hepatitis B vaccine c) Hepatitis B immune globulin is not administered because the first dose of Hepatitis B vaccine is sufficient. d) A second dose of hepatitis B vaccine is given in lieu of Hepatitis B immune globulin.

a) Hepatitis B immune globulin 0.5ml is given along with the first dose of Hepatitis B vaccine Those pregnant women who are positive for HBsAg should receive HBV viral load testing and should be treated with antiviral medications in the third trimester if their HBV viral load is high. Their newborns should receive HBV vaccine and HBIG within 12 hours of birth to optimally prevent maternal-child HBV transmission.

Which contact precautions are necessary for a client with hepatitis A? a) The client needs a private room with the door closed b) Wear a gown and gloves when handling items contaminated by urine and feces c) Use a gown and gloves when handing the client's dishes and utensils. d) Use a gown, mask, and gloves for all people entering the room.

b. Hepatitis A is transmitted via the fecal-oral route; contact precautions must be used when there are articles that potential fecal or urine contamination. Neither a private room nor closed door is required; these are necessary only for respiratory (airborne) precautions. Hepatitis A is not transmitted via the airborne route and therefore a mask is not necessary; a gown and gloves are required only one handling articles that may be contaminated. Wearing gowns and gloves only one handling the client soiled linen, dishes, or utensils is too limited; a gown and gloves also should be worn when handling other fecally contaminated articles, such as a bedpan or rectal thermometer.

The patient with advanced cirrhosis asks why his abdomen is so swollen. The nurse's response is based on the knowledge that a. a lack of clotting factors promotes the collection of blood in the abdominal cavity. b. portal hypertension and hypoalbuminemia cause a fluid shift into the peritoneal space. c. decreased peristalsis in the GI tract contributes to gas formation and distention of the bowel. d. bile salts in the blood irritate the peritoneal membranes, causing edema and pocketing of fluid.

b. Rationale: Ascites is the accumulation of serous fluid in the peritoneal or abdominal cavity and is a common manifestation of cirrhosis. With portal hypertension, proteins shift from the blood vessels through the larger pores of the sinusoids (capillaries) into the lymph space. When the lymphatic system is unable to carry off the excess proteins and water, those substances leak through the liver capsule into the peritoneal cavity. Osmotic pressure of the proteins pulls more fluid into the peritoneal cavity. A second mechanism of ascites formation is hypoalbuminemia, which results from the inability of the liver to synthesize albumin. Hypoalbuminemia results in decreased colloidal oncotic pressure. A third mechanism is hyperaldosteronism, which occurs when aldosterone is not metabolized by damaged hepatocytes. The increased aldosterone level causes increases in sodium reabsorption by the renal tubules. Sodium retention and an increase in antidiuretic hormone levels cause further water retention.

A patient has been told that she has elevated liver enzymes caused by nonalcoholic fatty liver disease (NAFLD). The nursing teaching plan should include a. having genetic testing done. b. recommending a heart-healthy diet. c. the necessity to reduce weight rapidly. d. avoiding alcohol until liver enzymes return to normal.

b. Rationale: Nonalcoholic fatty liver disease (NAFLD) can progress to liver cirrhosis. There is no definitive treatment, and therapy is directed at reduction of risk factors, which include treatment of diabetes, reduction in body weight, and elimination of harmful medications. For patients who are overweight, weight reduction is important. Weight loss improves insulin sensitivity and reduces liver enzyme levels. No specific dietary therapy is recommended. However, a hearthealthy diet as recommended by the American Heart Association is appropriate.

The primary diagnostic tests for acute pancreatitis are _________, which will be ________ in these individuals a. Serum amylase and lipase; low b. Serum amylase and lipase; high c. Serum trypsin and chymotrypsin; high d. Bile and Serum amyloid; high

b. Serum amylase and lipase; high Confirming the diagnosis of chronic pancreatitis can be hard. The diagnosis is based on the patient's signs and symptoms, laboratory studies, and imaging. In chronic pancreatitis, serum amylase and lipase levels may be increased slightly or not at all, depending on the degree of pancreatic fibrosis. Serum bilirubin and alkaline phosphatase levels may be increased. There is usually mild leukocytosis and a high sedimentation rate. ERCP can visualize the pancreatic and common bile ducts. Imaging studies, such as CT, MRI, MRCP, abdominal ultrasound, and EUS, can show a variety of changes, including calcifications, ductal dilation, pseudocysts, and enlargement of the pancreas. Stool samples are examined for fecal fat content. Deficiencies of fat-soluble vitamins and cobalamin, glucose intolerance, and diabetes may occur in those with chronic pancreatitis. A secretin stimulation test can assess the degree of pancreatic dysfunction.

Teaching in relation to home management after a laparoscopic cholecystectomy should include a. keeping the bandages on the puncture sites for 48 hours. b. reporting any bile-colored drainage or pus from any incision. c. using over-the-counter antiemetics if nausea and vomiting occur. d. emptying and measuring the contents of the bile bag from the T tube every day.

b. reporting any bile-colored drainage or pus from any incision.

A client presents to the emergency department with a history of IV drug use and homelessness. The client's lab work shows anti-HAV and IgG. Based off this labwork, the nurse knows that a) The client has an active Hepatitis B infection. b) The client has an active Hepatitis A infection c) The client had a previous Hepatitis A infection and has developed immunity to the virus. d) This is indicative of the preicteric phase of Hepatitis

c The presence of anti-HAV (IgG or IgM) in human serum or plasma is indicative of past or present infection with hepatitis A virus (HAV) or vaccination against HA IgM is most useful for determining recent infection (think iMMediate!), it usually becomes undetectable weeks to months following infection; in contrast, IgG is usually detectable for longer periods

A client returns to the progressive unit after a cholecystectomy. The client has a choledochostomy with a T-tube connected to gravity drainage. What is the function of the T-tube? a, The T-tube is used for insertion of antibiotics directly to the surgical site b. The T-tube allows for the irrigation of the common bile duct c The T-tube allows drainage of bile while swelling goes down d. The T-tube is used to help reform the gallbladder after surgery.

c.

Which nursing intervention is necessary in order to prevent stimulating the pancreas in a client admitted to the progressive floor with acute pancreatitis? a. Ensure the gastric tube is in the fundus of the stomach b. Maintain the gastric pH at a level less than 3.1 c. Administer the histamine H2 receptor antagonist that is in the client's MAR d. Consult physical therapy to ambulate the client 4 times a day

c. Administer the histamine H2 receptor antagonist that is in the client's MAR

__________ is s sign of acute hemorrhagic pancreatitis, this is ecchymoses of the flank area; wheras ___________ is s sign of acute hemorrhagic pancreatitis, this is ecchymoses of the periumbilical area. a. Babinski sign; Slate-Gray sign b. Battle sign; Levine's sign c. Grey Turner's sign; Cullen's sign d. Kernig sign; Brudzinski sign

c. Grey Turner's sign; Cullen's sign

A patient with pancreatic cancer is admitted to the hospital for evaluation of treatment options. The patient asks the nurse to explain the Whipple procedure that the surgeon has described. The explanation includes the information that a Whipple procedure involves a. creating a bypass around the obstruction caused by the tumor by joining the gallbladder to the jejunum. b. resection of the entire pancreas and the distal part of the stomach, with anastomosis of the common bile duct and the stomach into the duodenum. c. removal of part of the pancreas, part of the stomach, the duodenum, and the gallbladder, with joining of the pancreatic duct, the common bile duct, and the stomach into the jejunum. d. removal of the pancreas, the duodenum, and the spleen, and attachment of the stomach to the jejunum, which requires oral supplementation of pancreatic digestive enzymes and insulin replacement therapy.

c. Rationale: The classic operation for pancreatic cancer is a radical pancreaticoduodenectomy, or Whipple procedure. This entails resection of the proximal pancreas (i.e., proximal pancreatectomy), the adjoining duodenum (i.e., duodenectomy), the distal part of the stomach (i.e., partial gastrectomy), and the distal segment of the common bile duct. The pancreatic duct, common bile duct, and stomach are anastomosed to the jejunum.

A client who reports malaise, fever, abdominal pain, and is slightly jaundiced. The client has just returned from a trip to a developing country and was helping establish safe drinking water for the locals. Which intervention would be the most likely one needed for the client? a) The client needs a referral for hepatitis B immune globulin administration b) The client needs a QuantiFeron-TB blood test to assess for M. tuberculosis infection c) The client needs a referral for administration of immunoglobin (IG) to prevent a hepatitis A infection. ? d) The client needs to rest, drink plenty of fluids, and use NSAIDS until symptoms reside.

d, No specific treatment exists for hepatitis A. Your body will clear the hepatitis A virus on its own. In most cases of hepatitis A, the liver heals within six months with no lasting damage.

Which type of hepatitis develops from a chronic form of the disease? a) Hepatitis B and E b) Hepatitis A and B c) Hepatitis A d) Hepatitis B

d. -A is not chronic, B can be chronic (more common in infants and <5), C can be chronic (common), D can be chronic (depends on B to survive), E chronic only in immunocompromised patients -Some HBV and most HCV result in chronic hepatitis


Conjuntos de estudio relacionados

PEDS: Chapter 13 Key Pediatric Nursing Interventions

View Set

Carbohydrates,Proteins,Lipids Nucleic Acids

View Set

Organization Leadership Finals Preview

View Set

ASCP Exam Simulator Missed Questions

View Set

Econ 330 - Chapter 2: Financial Markets

View Set

Chapter 20: Alterations in Respiratory Function

View Set

IS 3513 Exam One (Modules 1, 2, 3)

View Set

Uiowa Quest for Human Destiny Final Exam

View Set

Section 1 Part 2 ReviewWhich of the following is an example of a buffer

View Set